What Does ROC Mean in Signals and Processing?

  • Thread starter Thread starter jaus tail
  • Start date Start date
  • Tags Tags
    Mean
Click For Summary
The discussion focuses on finding the Laplace transform of e^(-at)u(t) where a > 0, resulting in X(s) = 1/(s+a). The concept of Region of Convergence (ROC) is introduced, emphasizing that for the integral to converge, s must be greater than -a. This ensures that the exponent in the integral remains negative, allowing the transform to be defined. The ROC indicates the values of s for which the Laplace transform is valid. Understanding ROC is crucial in signals and processing to determine the stability and behavior of systems.
jaus tail
Messages
613
Reaction score
48

Homework Statement


Find laplace of e-atu(t) where a > 0

Homework Equations


Laplace is integration from -inf to +inf f(t)e-stdt
u(t) is 1 for t more than equal to 0.

The Attempt at a Solution


Well i got the answer as X(s) = 1/(s+a).

But the book said something like ROC like region of convergence and for that s must be more than -a
I understand that s more than minus a, gives positive 1/(s+a)
but why?

Why can't ROC be negative? What does ROC mean? I am studying Signals and Processing.
 
Physics news on Phys.org
jaus tail said:

Homework Statement


Find laplace of e-atu(t) where a > 0

Homework Equations


Laplace is integration from -inf to +inf f(t)e-stdt
u(t) is 1 for t more than equal to 0.

The Attempt at a Solution


Well i got the answer as X(s) = 1/(s+a).

But the book said something like ROC like region of convergence and for that s must be more than -a
I understand that s more than minus a, gives positive 1/(s+a)
but why?

Why can't ROC be negative? What does ROC mean? I am studying Signals and Processing.
Your integral is ##\int_1^\infty e^{-st}e^{-at}~dt = \int_1^\infty e^{-(s+a)t}~dt##. That integral will only converge if the exponent is negative, meaning ##s+a>0##. ROC in this case means the region of convergence, the values of ##s## for which the transform is defined.
 
  • Like
Likes jaus tail
Question: A clock's minute hand has length 4 and its hour hand has length 3. What is the distance between the tips at the moment when it is increasing most rapidly?(Putnam Exam Question) Answer: Making assumption that both the hands moves at constant angular velocities, the answer is ## \sqrt{7} .## But don't you think this assumption is somewhat doubtful and wrong?

Similar threads

  • · Replies 6 ·
Replies
6
Views
5K
  • · Replies 4 ·
Replies
4
Views
2K
  • · Replies 2 ·
Replies
2
Views
2K
  • · Replies 1 ·
Replies
1
Views
3K
  • · Replies 2 ·
Replies
2
Views
2K
Replies
2
Views
951
  • · Replies 1 ·
Replies
1
Views
1K
Replies
9
Views
2K
  • · Replies 1 ·
Replies
1
Views
3K
  • · Replies 1 ·
Replies
1
Views
5K